Difference between revisions of "2015 AMC 10B Problems/Problem 23"
m (→Solution 4) |
(→Solution 3: use \left and \right to make brackets bigger) |
||
Line 33: | Line 33: | ||
Let <math>n=5m+k</math> for some natural numbers <math>m</math>, <math>k</math> such that <math>k\in\{0,1,2,3,4\}</math>. Notice that <math>n<5^3=125</math>. Thus | Let <math>n=5m+k</math> for some natural numbers <math>m</math>, <math>k</math> such that <math>k\in\{0,1,2,3,4\}</math>. Notice that <math>n<5^3=125</math>. Thus | ||
− | <cmath>3(\lfloor\frac{n}{5}\rfloor+\lfloor\frac{n}{25}\rfloor)=\lfloor\frac{2n}{5}\rfloor+\lfloor\frac{2n}{25}\rfloor+\lfloor\frac{2n}{125}\rfloor</cmath> | + | <cmath>3(\left\lfloor\frac{n}{5}\right\rfloor+\left\lfloor\frac{n}{25}\right\rfloor)=\left\lfloor\frac{2n}{5}\right\rfloor+\left\lfloor\frac{2n}{25}\right\rfloor+\left\lfloor\frac{2n}{125}\right\rfloor</cmath> |
− | For smaller <math>n</math>, we temporarily let <math>\lfloor\frac{2n}{125}\rfloor=0</math> | + | For smaller <math>n</math>, we temporarily let <math>\left\lfloor\frac{2n}{125}\right\rfloor=0</math> |
− | <cmath>3(\lfloor\frac{n}{5}\rfloor+\lfloor\frac{n}{25}\rfloor)=\lfloor\frac{2n}{5}\rfloor+\lfloor\frac{2n}{25}\rfloor</cmath> | + | <cmath>3(\left\lfloor\frac{n}{5}\right\rfloor+\left\lfloor\frac{n}{25}\right\rfloor)=\left\lfloor\frac{2n}{5}\right\rfloor+\left\lfloor\frac{2n}{25}\right\rfloor</cmath> |
− | <cmath>3(\lfloor\frac{5m+k}{5}\rfloor+\lfloor\frac{5m+k}{25}\rfloor)=\lfloor\frac{2(5m+k)}{5}\rfloor+\lfloor\frac{2(5m+k)}{25}\rfloor</cmath> | + | <cmath>3(\left\lfloor\frac{5m+k}{5}\right\rfloor+\left\lfloor\frac{5m+k}{25}\right\rfloor)=\left\lfloor\frac{2(5m+k)}{5}\right\rfloor+\left\lfloor\frac{2(5m+k)}{25}\right\rfloor</cmath> |
− | <cmath>3(\lfloor\frac{5m+k}{5}\rfloor+\lfloor\frac{5m+k}{25}\rfloor)=\lfloor\frac{10m+2k}{5}\rfloor+\lfloor\frac{10m+2k}{25}\rfloor</cmath> | + | <cmath>3(\left\lfloor\frac{5m+k}{5}\right\rfloor+\left\lfloor\frac{5m+k}{25}\right\rfloor)=\left\lfloor\frac{10m+2k}{5}\right\rfloor+\left\lfloor\frac{10m+2k}{25}\right\rfloor</cmath> |
− | <cmath>3m+3\lfloor\frac{5m+k}{25}\rfloor=2m+\lfloor\frac{2k}{5}\rfloor+\lfloor\frac{10m+2k}{25}\rfloor</cmath> | + | <cmath>3m+3\left\lfloor\frac{5m+k}{25}\right\rfloor=2m+\left\lfloor\frac{2k}{5}\right\rfloor+\left\lfloor\frac{10m+2k}{25}\right\rfloor</cmath> |
− | <cmath>m+3\lfloor\frac{5m+k}{25}\rfloor=\lfloor\frac{2k}{5}\rfloor+\lfloor\frac{10m+2k}{25}\rfloor</cmath> | + | <cmath>m+3\left\lfloor\frac{5m+k}{25}\right\rfloor=\left\lfloor\frac{2k}{5}\right\rfloor+\left\lfloor\frac{10m+2k}{25}\right\rfloor</cmath> |
− | To minimize <math>n</math>, we let <math>\lfloor\frac{5m+k}{25}\rfloor=\lfloor\frac{10m+2k}{25}\rfloor=0</math>, then | + | To minimize <math>n</math>, we let <math>\left\lfloor\frac{5m+k}{25}\right\rfloor=\left\lfloor\frac{10m+2k}{25}\right\rfloor=0</math>, then |
− | <cmath>m=\lfloor\frac{2k}{5}\rfloor</cmath> | + | <cmath>m=\left\lfloor\frac{2k}{5}\right\rfloor</cmath> |
Since <math>k<5</math>, <math>m>0</math>, the only integral value of <math>m</math> is <math>1</math>, from which we have <math>k=3,4\Longrightarrow n=8,9</math>. | Since <math>k<5</math>, <math>m>0</math>, the only integral value of <math>m</math> is <math>1</math>, from which we have <math>k=3,4\Longrightarrow n=8,9</math>. | ||
− | Now we let <math>\lfloor\frac{5m+k}{25}\rfloor=0</math> and <math>\lfloor\frac{10m+2k}{25}\rfloor=1</math>, then | + | Now we let <math>\left\lfloor\frac{5m+k}{25}\right\rfloor=0</math> and <math>\left\lfloor\frac{10m+2k}{25}\right\rfloor=1</math>, then |
− | <cmath>m=\lfloor\frac{2k}{5}\rfloor+\lfloor\frac{10m+2k}{25}\rfloor</cmath> | + | <cmath>m=\left\lfloor\frac{2k}{5}\right\rfloor+\left\lfloor\frac{10m+2k}{25}\right\rfloor</cmath> |
Since <math>k<5</math>, <math>10m>15\Longrightarrow m\ge2</math>. | Since <math>k<5</math>, <math>10m>15\Longrightarrow m\ge2</math>. | ||
If <math>m>2</math>, then | If <math>m>2</math>, then | ||
− | <cmath>m>\lfloor\frac{2k}{5}\rfloor+\lfloor\frac{10m+2k}{25}\rfloor</cmath> | + | <cmath>m>\left\lfloor\frac{2k}{5}\right\rfloor+\left\lfloor\frac{10m+2k}{25}\right\rfloor</cmath> |
which is a contradiction. | which is a contradiction. | ||
− | Thus <math>m=2\Longrightarrow\lfloor\frac{2k}{5}\rfloor=1\Longrightarrow n=13,14</math> | + | Thus <math>m=2\Longrightarrow\left\lfloor\frac{2k}{5}\right\rfloor=1\Longrightarrow n=13,14</math> |
Finally, the sum of the four smallest possible <math>n=8+9+13+14=44</math> and <math>4+4=8</math>. <math>\boxed{\mathrm{(B)}}</math> | Finally, the sum of the four smallest possible <math>n=8+9+13+14=44</math> and <math>4+4=8</math>. <math>\boxed{\mathrm{(B)}}</math> |
Revision as of 11:41, 24 August 2021
Problem
Let be a positive integer greater than 4 such that the decimal representation of ends in zeros and the decimal representation of ends in zeros. Let denote the sum of the four least possible values of . What is the sum of the digits of ?
Solution 1
A trailing zero requires a factor of two and a factor of five. Since factors of two occur more often than factors of five, we can focus on the factors of five. We make a chart of how many trailing zeros factorials have:
We first look at the case when has zero and has zeros. If , has only zeros. But for , has zeros. Thus, and work.
Secondly, we look at the case when has zeros and has zeros. If , has only zeros. But for , has zeros. Thus, the smallest four values of that work are , which sum to . The sum of the digits of is
Solution 2
By Legendre's Formula and the information given, we have that .
We have as there is no way that if , would have times as many zeroes as .
First, let's plug in the number . We get that , which is obviously not true. Hence,
After several attempts, we realize that the RHS needs to more "extra" zeroes than the LHS. Hence, is greater than a multiple of .
We find that the least four possible are .
.
Solution 3
Let for some natural numbers , such that . Notice that . Thus For smaller , we temporarily let To minimize , we let , then Since , , the only integral value of is , from which we have .
Now we let and , then Since , .
If , then which is a contradiction.
Thus
Finally, the sum of the four smallest possible and .
~ Nafer
See Also
2015 AMC 10B (Problems • Answer Key • Resources) | ||
Preceded by Problem 22 |
Followed by Problem 24 | |
1 • 2 • 3 • 4 • 5 • 6 • 7 • 8 • 9 • 10 • 11 • 12 • 13 • 14 • 15 • 16 • 17 • 18 • 19 • 20 • 21 • 22 • 23 • 24 • 25 | ||
All AMC 10 Problems and Solutions |
The problems on this page are copyrighted by the Mathematical Association of America's American Mathematics Competitions.